LSAT and Law School Admissions Forum

Get expert LSAT preparation and law school admissions advice from PowerScore Test Preparation.

 PatMcGroin
  • Posts: 5
  • Joined: Nov 02, 2020
|
#83184
I decided on B as the answer choice b/c I thought it was the methodology type question, where the conclusion can be right even if there was something wrong with the poll/survey. Could you please explain what kind of flaw the correct answer choice (i.e. A) suggests?
 Jeremy Press
PowerScore Staff
  • PowerScore Staff
  • Posts: 1000
  • Joined: Jun 12, 2017
|
#83360
Hi there,

The problem with answer choice B is that the possibility of the survey being "accepted by the public" doesn't have any effect on whether the survey's conclusion (that "most people in the country favor investing more money in information technologies") is false, which is the main point of the argument. In other words, whether or not the public accepts the results of a survey is not the same issue as whether or not the result of the survey itself is true or false. That makes answer choice B irrelevant.

Answer choice A isn't describing a technical logical fallacy of a type you might learn from a "Flaw list" like the one we provide in the Logical Reasoning Bible or in Lesson 7 of our online courses. It's just describing a possibility that would be problematic for the conclusion of the argument. This is something we're seeing more of these days on Flaw questions (answers that raise problematic possibilities that the argument hasn't ruled out). What's the possibility answer choice A is describing? It's the possibility that, even if you take out the falsified data, the survey might actually still turn out to show (with the remaining "good" data) that most people favor investing more money in info tech. If that were true, then we couldn't conclude, as the author of the argument does, that the survey shows "a majority does not favor more investment in information technologies." Since the possibility described in answer choice A weakens the argument, it makes the argument "vulnerable to criticism," and is the correct answer.

I hope this helps!

Get the most out of your LSAT Prep Plus subscription.

Analyze and track your performance with our Testing and Analytics Package.